Vous êtes sur la page 1sur 52

Lectures On Fourier Series

S. Kesavan
Institute of Mathematical Sciences
Chennai-600 113, INDIA

Third Annual Foundational School - Part I


December 4–30, 2006
Contents
1 Introduction 3

2 Orthonormal Sets 6

3 Variations on the Theme 11

4 The Riemann-Lebesgue Lemma 12

5 The Dirichlet, Fourier and Fejér Kernels 15

6 Fourier Series of Continuous Functions 23

7 Fejér’s Theorem 27

8 Regularity 30

9 Pointwise Convergence 39

10 Termwise Integration 44

11 Termwise Differentiation 47

References 52

2
1 Introduction
With the invention of calculus by Newton (1642–1727) and Leibnitz (1646–
1716), there was a surge of activity in various topics of mathematical physics,
notably in the study of boundary value problems associated to vibrations of
strings stretched between points and vibration of bars or columns of air asso-
ciated with mathematical theories of musical vibrations. Early contributors
to the theory of vibrating strings include B. Taylor (1685–1731), D. Bernoulli
(1700–1782), L. Euler (1707–1783) and d’Alembert (1717–1783).
By the middle of the eighteenth century, d’Alembert, Bernoulli and Euler
had advanced the theory of vibrating strings to the stage where the partial
differential eqaution (now called the wave equation)

∂2y 2
2∂ y
= a
∂t2 ∂x2
was known and a solution of the boundary value problem had been found
from the general solution of that equation. The concept of fundamental
modes of vibration led them to the notion of superposition of solutions and
Bernoulli proposed a solution of the form

X nπx nπat
y(x, t) = bn sin cos (1.1)
n=1
c c

(where c is the length of the string). The initial position of the string f (x)
will then be represented by

X nπx
f (x) = bn sin (0 ≤ x ≤ c). (1.2)
n=1
c

Later, Euler gave the formulas for the coefficients bn .


But the general concept of a function had not been clarified and a lengthy
controversy took place over the question of representing arbitrary functions
on a finite interval by a series of sine functions.
The French mathematician d’Alembert gave an elegant solution in the
form
y(x, t) = v(at + x) − v(at − x), (1.3)
and he believed that he had solved the problem completely.

3
At that time, the word function had a very restricted meaning and was
understood as something given by an analytic expression. Euler thought that
the initial position of a plucked string need not always be a function, but
some form where different parts could be expressed by different functions. In
other words, function and graph meant different things. For every function,
we can draw its graph but every graph that can be drawn need not come
from a function.
Euler strongly objected to Bernoulli’s claim that every solution to the
problem of a plucked string could be represented in the form (1.1), on two
counts. First of all the right-hand side of (1.1) was a periodic function, while
the left-hand side was arbitrary. Further, the right-hand side of (1.2) was
an analytic formula and hence a function, while the left-hand side, f , could
be any graph. So Euler believed that d’Alembert’s solution was valid when
f was any graph while Bernoulli’s solution was applicable only to a very
restricted class of functions.
J. B. Fourier (1768–1830) presented many instructive examples of expan-
sions of functions in trigonometric series in connection with boundary value
problems associated to the conduction of heat. His book “Théorie Analytique
de la Chaleur ” (1822) is a classic. Fourier never justified the convergence of
his series expansions and this was objected to by his contemporaries La-
grange, Legendre and Laplace. Fourier asserted that any periodic function
could be written as a trigonometric series.
Dirichlet (1805–1859) firmly established in 1829 (nearly seventy years
after the controversy started), sufficient conditions on a function f so that
its Fourier series converges to its value at a point.
Since then a lot of ideas and theories grew out of a need to understand
what these series meant. Amongst them are Cantor’s theory of infinite sets,
the rigorous notion of a function, the theories of integration due to Riemann
and Lebesgue and the theories of summability of series.
In mathematical analysis, we always try to find approximation of objects
by simpler objects. For example, we approximate real numbers by rationals.
By truncating the Taylor series of a function, we approximate the function by
a polynomial. However, for a function to admit a Taylor series, it has to be
infinitely differentiable (but this is not sufficient!) in some interval but this is
quite restrictive. Indeed Weierstrass’ approximation theorem states that any
continuous function defined on a finite closed interval can be approximated
uniformly by a polynomial.
Now, consider the set of all functions {1} ∪ {cos nt, sin nt | n ∈ N} on

4
[−π, π]. Given any point t ∈ [−π, π], the constant function does not vanish
at t. Further if t1 and t2 are distinct points in [−π, π] then we can always
find a function in the above set such that it takes different values at t1 and
t2 . Thus, the set of all trigonometric polynomials, viz. functions of the form
N
X
f (t) = a0 + (an cos nt + bn sin nt) (1.4)
n=1

form an algebra (i.e. the set is closed under pointwise addition, multipli-
cation and scalar multiplication) and it does not ‘vanish at any point’ and
‘separates points’. By the Stone-Weierstrass theorem (which generalizes the
Weierstrass’ approximation theorem), every periodic continuous function on
[−π, π] can be approximated uniformly by trigonometric polynomials.
A trigonometric polynomial of the form (1.4) can also be written in ex-
ponential form:
XN
f (x) = cn exp(inx). (1.5)
n=−N
It is easy to see that a0 = c0 , b0 = 0 and that
an = cn + c−n ; bn = i(cn − c−n )
or, equivalently,
an − ibn an + ibn
cn = ; c−n = .
2 2
If n is a non-zero integer, then exp(inx) is the derivative of exp(inx)/in,
which also has period 2π. Thus
Z π (
1 1 if n = 0
exp(inx) dx = (1.6)
2π −π 0 if n 6= 0.
Mulitplying (1.5) by exp(−imx) and integrating over [−π, π], we get, in view
of (1.6), Z π
1
cm = f (x) exp(−imx) dx. (1.7)
2π −π
This gives us, for any positive integer m,
1 π
Z 
am = f (x) cos mx dx 

π −π 
Z π (1.8)
1
bm = f (x) sin mx dx.


π −π

5
It is common practice to replace a0 by a0 /2, so that (1.8) is valid for a0 as
well.
We now generalize this to define the trigonometric series

X
exp(inx),
n=−∞

or, equivalently,

a0 X
+ (an cos nx + bn sin nx) .
2 n=1

Given a 2π-periodic function f on [−π, π], we define an (n ≥ 0) and bn


(n ≥ 1) by (1.8) and the resulting series is called the Fourier series of the
function f . The an and bn are called the Fourier coefficients of f .
The basic question now is when does the Fourier series of a function
converge? If it converges, does it converge to the value of f at the given
point? In other words, to what extent does the Fourier series of a function
represent the function itself?
In the sequel we will try and answer some of these questions.
To begin with, we will look at an abstract situation suggested by the
relations (1.6).

2 Orthonormal Sets
Let H be a Hilbert Space (over R or C). We denote the inner product in H
by (·, ·) and the norm it generates by k · k.

Definition 2.1. Let S = {ui | i ∈ I}, where I is an indexing set, be a


collection of elements in H. The set S is said to be orthonormal if
)
kuik = 1 for all i ∈ I
(2.1)
(ui, uj ) = 0 for all i 6= j, i, j ∈ I.

Example 2.1. The standard basis {ei }1≤i≤n in Rn , where ei has 1 in the
ith -coordinate and zero elsewhere, is orthonormal in Rn with the usual inner-
product and the euclidean norm.

6
Example 2.2. Consider the space of square summable sequences `2 , i.e.
( ∞
)
X
`2 = x = (xi ) | |xi |2 < ∞ .
i=1

Then again, the set of sequences {en }∞ th


n=1 where en has 1 as the n entry and
zero at all other places, is orthonormal in `2 .

Example 2.3. The sequence { 2 sin nπx} is orthonormal in L2 (0, 1).
Example 2.4. The sequence { √12π } ∪ { sin√πnx , cos
√ nx | n ∈ N} is orthonormal in
π
L2 (−π, π).
Proposition 2.1. Let H be a separable Hilbert space. Then any orthonormal
set is at most countable.
Proof. Let {xn } be a countable dense set in √ H. If u and v are elements
in an orthonormal
√ set, we have ku − vk = 2. Thus each of the balls
Bn = B(xn ; 2/4) can contain at most one element of an orthonormal set.
Since the {xn } form a dense set, every member of H must belong to one such
ball. Hence the result.
Henceforth, we will assume that H is a separable Hilbert space over R.
Proposition 2.2. Let {e1 , . . . , en } be a finite orthonormal set in a Hilbert
space H. Then, for any x ∈ H, we have
n
X
|(x, ei )|2 ≤ kxk2 . (2.2)
i=1
Pn
Proof. We have kx − i=1 (x, ei )ei k2 ≥ 0. Expanding this, we get, using the
fact that the {ei } are orthonormal,
n
X n
X
2
2
kxk + |(x, ei )| − 2 |(x, ei )|2 ≥ 0
i=1 i=1

which proves (2.2).


Theorem 2.1 (Bessel’s Inequality). If {ei } is an orthonormal set in a
Hilbert Space H, then X
|(x, ei )|2 ≤ kxk2 . (2.3)
i

7
Proof. Since H is separable, {ei } is at most countable. The result, in the
finite case, has already been shown. If {ei } is countably infinite, then for each
n, we have that (2.2) is valid. Thus, since the result is true for all partial
sums, it is true for the series as well and we get (2.3).
Corollary 2.1. If {en } is an orthonormal sequence in H, then for every
x ∈ H, (x, en ) → 0 as n → ∞.
It is immediate to see that the elements of an orthonormal set are lin-
early independent. Further, given a set of linearly independent elements
{x1 , . . . , xn } in H, we can produce an orthonormal set {e1 , . . . , en } such that
the linear spans of {x1 , . . . , xk } and {e1 , . . . , ek } coincide for all 1 ≤ k ≤ n.
Indeed, set e1 = x1 /kx1 k. Define
x2 − (x2 , e1 )e1
e2 = .
kx1 − (x2 , e1 )e1 k
(Notice, by the linear independence of x1 and x2 , the vector x2 − (x2 , e1 )e1
cannot be zero.) It is easy to see that ke2 k = 1 and that (e1 , e2 ) = 0. In
general, assume that we have constructed e1 , . . . , ek such that
(i) each ei (1 ≤ i ≤ k) is a linear combination of x1 , . . . , xi and
(ii) the {ei } are orthonormal.
Now define Pk
xk+1 − (xk+1 , ei )ei
ek+1 = Pki=1 .
kxk+1 − i=1 (xk+1 , ei )ei k
Thus, inductively we obtain {e1 , . . . , en }. This procedure is called the Gram-
Schmidt orthogonalization procedure.
Thus, if H is a finite dimensional space, we can construct an orthonormal
basis for H.
Henceforth, we will assume that H is infinite dimensional and separable.
Definition 2.2. An orthonormal set is complete if it is maximal with respect
to the partial ordering on orthonormal sets in H induced by inclusion.
Let {en } be an orthonormal sequence in an infinite dimensional (separa-
ble) Hilbert space. Let x ∈ H. Define
n
X
yn = (x, en )en .
i=1

8
Then, for m > n,
m
X
2
kyn − ym k = |(x, ei )|2
i=n+1

and the right-hand side tends to zero, by Bessel’s inequality. Thus, {yn } is a
Cauchy sequence and so converges in H. We define the limit to be

X
(x, ei )ei .
i=1

Proposition 2.3. The vector x − ∞


P
j=1 (x, ej )ej is orthogonal to each ei .
Further,

X X ∞
2 2
kx − (x, ei )ei k = kxk − |(x, ei )|2 . (2.4)
i=1 i=1

Proof. Given any n, set yn = i=1 (x, ej )ej . Then yn → ∞


Pn P
i=1 (x, ei )ei in H.
Now, if 1 ≤ i ≤ n, clearly (x − yn , ei ) = 0. Fix i, and the above relation
holds for all n ≥ i. Thus,
X
(x − (x, ej )ej , ei ) = 0.
j

Now,
n
X n
X
2 2 2
kx − yn k = kxk + |(x, ei )| − 2 |(x, ei )|2
i=1 i=1
Xn
= kxk2 − |(x, ei )|2 .
i=1

Thus, passing to the limit as n → ∞, we get (2.4).


Theorem 2.2. Let H be a Hilbert space and {ei } an orthonormal set in H.
The following are equivalent.
(i) {ei } is complete.
(ii) If x ∈ H such that (x, ei ) = 0 for all i, then x = 0.
(iii) If x ∈ H, then X
x= (x, ej )ej . (2.5)
j

9
(iv) (Parseval’s Identity) If x ∈ H, then
X
kxk2 = |(x, ei )|2 . (2.6)
i

n o
x
Proof. (i)⇒ (ii) If (x, ei ) = 0 for all i and x 6= 0, then {ei } ∪ will bekxk
an orthonormal set contradicting the Pmaximality of {ei }.
(ii)⇒ (iii) By Proposition 2.3, x − j (x, ej )ej is orthogonal to each ei and
so, by (ii) we get (2.5).
(iii)⇒ (iv) This is an immediate consequence of (2.4).
(iv)⇒ (i) If {ei } were not maximal, there exists e ∈ H such that kek = 1
and (e, ei ) = 0 for all i. This contradicts (2.6).

Corollary 2.2. An orthonormal set {ei } in a Hilbert space H is complete,


if, and only if, the linear span of the {ei }, i.e. the space of all (finite) linear
combinations of the {ei }, is dense in H.

Proof. If {ei } is complete, then by (2.5) we get that each x ∈ H is such that
n
X
x = lim (x, ei )ei .
n→∞
i=1

Thus the linear span of {ei } is dense in H. Conversely, if the linear span is
dense, then, if x is orthogonal to all the ei , it follows that x = 0. Thus {ei }
is complete.

Remark 2.1. In view of (2.5), a complete orthonormal set is also called an


orthonormal basis.

Example 2.5. Consider the sequence {en } in `2 (cf. Example 2.2). This
sequence is complete in `2 since

X ∞
X
kxk2 = |xi |2 = |(x, ei )|2 .
i=1 i=1

Example 2.6. Consider the Hilbert space L2 (−π, π). It is known that contin-
uous functions with compact support are dense in this space. Such functions
are periodic (they vanish at −π and π) and we saw in §1 that, by virtue
of the Stone-Weierstrass theorem, they can be uniformly approximated by

10
trigonometric polynomials. It then follows, a fortiori, that they can also be
approximated in the L2 -norm. Thus, the trignometric polynomials are dense
in L2 (−π, π) and so, by the preceding corollary, the functions
   
1 cos nt sin nt
√ ∪ √ , √ |n∈N
2π π π

form a complete orthonormal set in L2 (−π, π). In particular, if


f ∈ L2 (−π, π), we have, by Parseval’s identity,
π Z π 2
1
Z
2
|f | dx = f (t) dt +
−π 2π −π

" Z 2 2 #
π Z π
X 1 1
+ f (t) cos nt dt + f (t) sin nt dt
n=1
π −π π −π

which yields
π ∞
1 a20 X
Z
|f |2 dx = |an |2 + |bn |2

+ (2.7)
π −π 2 n=1

where an , bn are the Fourier coefficients of f given by (1.8).


By analogy with the above example, if {ei } is an orthonormal basis for a
separable Hilbert space H, we say that the Fourier series of x is the series
P ∞
i=1 (x, ei )ei and the quantities (x, ei ) are called the Fourier coefficients.

3 Variations on the Theme


Let f be a 2π-periodic function defined on [−π, π]. The Fourier coefficients
of f are given by the formulas (1.8). These formulas also make sense if
f ∈ L1 (−π, π). By changing the values of a function at a finite number
of points, we do not change the values of the Fourier coefficients. (Indeed,
recall that the spaces Lp are only equivalence classes of functions, under the
equivalence relation given by f ∼ g if f = g a.e.; thus, it is meaningless to
talk of the value of a function in Lp at a particular point). Given a function
h such that h(π) 6= h(−π), we can redefine it so that h(π) = h(−π) and then
extend it periodically as a 2π-periodic function over R. We can also declare
the function to be undefined at −π or π. Thus we are forced to consider the
Fourier series of functions with jump discontinuities.

11
If a function belongs to L1 (0, π) then we can extend it either as an odd
function or as an even function to L1 (−π, π). In the former case, all the
coefficients an will be zero and we get a series only involving the functions
sin nt. This is called the Fourier sine series of the given function. Similarly,
in the latter case, only the coefficients an will be non-zero and the resulting
series, which involves only the functions cos nt, is called the Fourier cosine
series of the function.
We can also rewrite the Fourier series of a function in the amplitude-phase
form. Indeed, let

a0 X
+ (an cos nt + bn sin nt) (3.1)
2 n=1

be the Fourier series of a function. Set

an = dn cos φn , bn = dn sin φn .

In other words,
p
dn = a2n + b2n , φn = cos−1 (an /dn ).

Then the series (3.1) can be rewritten as


a0 X
+ dn cos(nt − φn ). (3.2)
2 n

4 The Riemann-Lebesgue Lemma


We have seen earlier that if {en } were an orthonormal sequence in a Hilbert
space H, then, for any x ∈ H, we have

lim (x, en ) = 0.
n→∞

In particular, if f ∈ L2 (−π, π) we deduce that


Z π Z π
lim f (t) cos nt dt = lim f (t) sin nt dt = 0.
n→∞ −π n→∞ −π

This result is one of the forms of what is called the Riemann-Lebesgue lemma.
We now prove a very useful generalization of this.

12
Theorem 4.1 (Generalized Riemann-Lebesgue Lemma). Let
f ∈ L1 (a, b), where −∞ ≤ a < b ≤ +∞. Let h be a bounded measurable
function defined on R, such that
1 c
Z
lim h(t) dt = 0. (4.1)
c→±∞ c 0

Then Z b
lim f (t)h(ωt) dt = 0. (4.2)
ω→∞ a

Proof. We extend f by zero outside (a, b) so that we can consider f ∈ L1 (R).


Let us consider an interval [c, d] ⊂ (0, ∞). Then
Z ∞ Z d
1 dω
Z
χ[c,d] h(ωt) dt = h(ωt) dt = h(t) dt
0 c ω cω
1 dω 1 cω
Z Z
= h(t) dt − h(t) dt
ω 0 ω 0
and, by hypothesis, both integrals tend to zero as ω → ∞. The result now
follows, by linearity, to all step functions made up of characteristic functions
of intervals. However such functions are dense in L1 (0, ∞). (Indeed contin-
uous functions with compact support are dense in L1 (0, ∞); such functions
are uniformly continuous and by partitioning the interval containing the sup-
port, we can approximate continuous functions with compact support by step
functions uniformly and hence, a fortiori, in L1 (0, ∞).)
R ∞ Let |h| ≤ M. Thus, if f ∈ L1 (0, ∞), then find a step function g such that
ε
0
|f − g| dx < 2M , for a given ε > 0. Now,
Z ∞ Z ∞ Z ∞

f (t)h(ωt) dt ≤ |f (t) − g(t)|h(ωt) dt + g(t)h(ωt) dt

0 0 0
Z ∞
ε
≤ + g(t)h(ωt) dt
2 0

and for ω large enough the secondR term can also be made to be less than
ε 0
2
. A similar argument holds for −∞ f (t)h(ωt) dt and this completes the
proof.
Corollary 4.1. If f ∈ L1 (a, b) then
Z b Z b
lim f (t) cos nt dt = lim f (t) sin nt dt = 0. (4.3)
n→∞ a n→∞ a

13
Proof.
Z c
1 1 1
cos t dt = (sin c) ≤

c c |c|
Z c 0
1 1 2
sin t dt = (1 − cos c) ≤
c
0 c |c|

and both tend to zero as |c| → ∞.


We nowP give an immediate application of this result. Given a conver-
gent series ∞ n=1 αn , we know that αn → 0 as n → ∞. We now ask if a
trigonometric series

a0 X
+ (an cos nt + bn sin nt)
2 n=1

is convergent, whether an → 0 and bn → 0 as n → ∞.

Theorem
P∞ 4.2 (Cantor-Lebesgue Theorem). If a trigonometric series
a0
2
+ n=1 (an cos nt + bn sin nt) converges on a set E whose (Lebesgue) mea-
sure is positive, then an → 0 and bn → 0.

Proof. Without loss of generality, we may assume that E has finitep measure.
We rewrite the trigonometric series as in (3.2) where dn = a2n + b2n and
φn = cos−1 (an /dn ). Since the series converges, it follows that for all t ∈ E,

dn cos(nt − φn ) → 0

as n → ∞. Assume that {dn } does not converge to zero. Then, there exists
ε > 0 and a subsequence {nk } such that

dnk ≥ ε > 0,

for all k. Then, it follows that cos(nk t − φnk ) → 0 as k → ∞ for all t ∈ E.


Since E is of finite measure, it follows, from the dominated convergence
theorem, that Z
cos2 (nk t − φnk ) dt → 0. (4.4)
E

14
Now cos2 (nk t − φnk ) = 21 [1 + cos 2(nk t − φnk )]. But then χE ∈ L1 (R) and so
Z Z
cos 2(nk t − φnk ) dt = χE (t) cos 2(nk t − φnk ) dt
E R
Z
= cos 2φnk χE (t) cos 2nk t dt +
RZ

+ sin 2φnk χE (t) sin 2nk t dt


R

and both the integrals on the right-hand side tend to zero by Corollary 4.1.
It then follows that
µ(E)
Z
cos2 (nk t − φnk ) dt → >0
E 2
which contradicts (4.4). This shows that dn → 0 and so an → 0 and bn →
0.

5 The Dirichlet, Fourier and Fejér Kernels


Let f ∈ L1 (−π, π) with Fourier series given by

a0 X
f (t) ∼ + (an cos nt + bn sin nt) (5.1)
2 n=1

where an , bn are the Fourier coefficients defined as in (1.8). We wish to


examine the convergence of this series. In particular, we will be interested,
in the sequel, to answers (positive or negative) to the following questions.
• Does the Fourier series converge at all points t ∈ [−π, π]?

• If it converges at t ∈ [−π, π], does it converge to f (t)?

• If it converges at all t ∈ [−π, π], is the convergence uniform?


To discuss the convergence, pointwise or uniform, of the Fourier series, we
need to discuss the convergence of the sequence {sn } of partial sums. We
have n
a0 X
sn (t) = + (ak cos kt + bk sin kt). (5.2)
2 k=1

15
Equivalently, in exponential form,
n
X
sn (t) = ck exp(ikt).
k=−n

Using the expression for the ck (cf. (1.7)) we get


n Z π
X 1
sn (t) = f (x) exp(ik(t − x)) dx
k=−n
2π −π
Z π
1
= f (x)Dn (t − x) dx
2π −π
where n
X
Dn (t) = exp(ikt). (5.3)
k=−n

The 2π-periodic function Dn (t) is called the Dirichlet Kernel.


Proposition 5.1. Let {sn } be the sequence of partial sums of the Fourier
series of f ∈ L1 (−π, π) which is 2π-periodic. Then
Z π
1
sn (t) = f (x)Dn (t − x) dx (5.4a)
2π −π
Z π
1
= f (t − x)Dn (x) dx (5.4b)
2π −π
Z π
1
= f (x)Dn (x − t) dx (5.4c)
2π −π
Z π
1
= f (t + x)Dn (x) dx (5.4d)
2π −π
Z π
1
= (f (t + x) + f (t − x))Dn (x) dx (5.4e)
2π 0
Proof. We have already established (5.4a). By a change of variable t − x = y
we get Z π+t
1
sn (t) = f (t − y)Dn (y) dy.
2π −π+t
By the 2π-periodicity, it follows that the integral does not change as long as
the length of the interval of integration is 2π. This proves (5.4b). The relation

16
(5.4c) follows from (5.4a) since Dn is easily seen to be an even function.
Relation (5.4d) follows from (5.4c), again by a change of variable y = x − t
and the fact that the integrals do not change as long as the length of the
interval is 2π. Finally, we split the integral in (5.4d) as the sum of integrals
over [−π, 0] and [0, π]. Now,
Z 0 Z π
f (t + x)Dn (x) dx = f (t − y)Dn (y) dy
−π 0

using the change of variable y = −x and the evenness of Dn . This proves


(5.4e).

Proposition 5.2. Let n ≥ 0 be an integer. Then



1
 sin(n + 2 )t t 6= 2kπ, k ∈ N ∪ {0}.

Dn (t) = sin 2t (5.5)
2n + 1 t = 2kπ, k ∈ N ∪ {0}.

Further,
π
1
Z
Dn (t) dt = 1. (5.6)
2π −π

Proof. When n = 2kπ, clearly Dn (t) = 2n+1. Assume n 6= 2kπ, k ∈ N∪{0}.


Then
(exp(it) − 1)Dn (t) = exp(i(n + 1)t) − exp(−int)
Multiplying both sides by exp(−it/2) we immediately deduce (5.5). The
relation (5.6) immediately follows from the definition of Dn (t) (cf. (5.3)) and
the relations (1.6).
We now introduce two other functions which, together with the Dirichlet
kernel, will play an important role in the study of the convergence of Fourier
series.

Definition 5.1. The continuous Fourier kernel is given by



 sin ωt
, t 6= 0
Φ(ω, t) = t (5.7)
ω, t=0

17
where ω and t are real numbers. The associated discrete Fourier kernel is
given by
1

 sin(n + 2 )t
t 6= 0
Φn (t) = t (5.8)
n + 12 t = 0,

where t ∈ R and n is a non-negative integer.


Remark 5.1. Clearly Dn is 2π-periodic. It is easy to see that sin(n+1/2)t
sin(t/2)

2n + 1 as t → ∞. Thus Dn is continuous. Similarly, it is easy to see that the
continuous and discrete Fourier kernels are also continuous.
Definition 5.2. The Fejér kernel is defined by
n
1 X
Kn (t) = Dk (t). (5.9)
n + 1 k=0

Proposition 5.3. Let n ≥ 0 be an integer. Then


1 1 − cos(n + 1)t
Kn (t) = (5.10)
n+1 1 − cos t
Z π
1
Kn (t) dt = 1. (5.11)
2π −π
Further, Kn ≥ 0 and if 0 < δ ≤ |t| ≤ π, we have
2
Kn (t) ≤ . (5.12)
(n + 1)(1 − cos δ)
Proof. As before, observe that
  
(n + 1)Kn (t) exp(it) − 1 exp(−it) − 1 = exp(−it) − 1
m 
X 

· exp i(k + 1)t − exp(−ikt)
k=0

= 2 − exp i(n + 1)t −

− exp −i(n + 1)t

from which we deduce (5.10). The relation (5.11) follows directly from the
definition (cf. (5.9)) and the relation (5.6). That Kn is non-negative follows
immediately from (5.10). So does relation (5.12).

18
We now derive some estimates for integrals of the Dirichlet and Fourier
kernels. First we need a technical result.

Lemma 5.1. Let {Ak } be a sequence of real numbers such that A2k−1 > 0
and A2k < 0 and such that |Ak+1 | < |Ak | for all k ∈ N. Then, for every
k ∈ N, we have
0 < A1 + . . . + Ak < A1 . (5.13)
Proof. Let k be odd. Then

A1 + (A2 + A3 ) + (A4 + A5 ) + . . . + (Ak−1 + Ak )

is such that each term in parentheses is negative. Thus the sum is less than
A1 . Again

(A1 + A2 ) + (A3 + A4 ) + . . . + (Ak−2 + Ak−1 ) + Ak

is such that each term in parentheses is greater than 0 and Ak > 0. Thus
the sum is greater than 0. This proves (5.13) in the case k is odd. The proof
when k is even is similar.
Proposition 5.4. Let 0 ≤ a < b ≤ π. Let n ≥ 0 be an integer. Then
Z b
sin(n + 1/2)t
dt ≤ 4π. (5.14)

a sin(t/2)
Proof. Let
kπ/(n+1/2)
sin(n + 1/2)t
Z
Ak = dt, 1 ≤ k ≤ n + 1.
(k−1)π/(n+1/2) sin(t/2)

Then in each such interval, the numerator of the integrand varies like sin t
between (k−1)π and kπ. On the other hand sin(t/2) is positive and increases.
Thus clearly hA1 > 0 and Ak alternatesh in signi and decreases in absolute value.
(k−1)π kπ nπ
Now let a ∈ n+1/2 , n+1/2 or a ∈ n+1/2 ,π .
 i

If a is in the interior of any of these intervals (or, if a ∈ n+1/2 , π , when
k = n + 1), we have
Z a
sin(n + 1/2)t
dt
(k−1)π/(n+1/2) sin(t/2)

19
is either greater than 0 (because the integrand is greater than 0, when k is
odd) or is less than 0 (k even) and
Z a
sin(n + 1/2)t
dt < |Ak |.

(k−1)π/(n+1/2) sin(t/2)

Thus
Z a a
sin(n + 1/2)t sin(n + 1/2)t
Z
dt = A1 + . . . + Ak−1 + dt
0 sin(t/2) (k−1)π/(n+1/2) sin(t/2)

and it follows that


a
sin(n + 1/2)t
Z
0< dt < A1 .
0 sin(t/2)

Now, in the interval [0, π/(n + 1/2)], the integrand is positive and decreasing
with maximum value (2n + 1) at t = 0. Thus
π
A1 < (2n + 1) = 2π.
(n + 1/2)

Now
b
Z Z b Z a
sin(n + 1/2)t sin(n + 1/2)t sin(n + 1/2)t
dt =
dt − dt

a sin(t/2) 0 sin(t/2) 0 sin(t/2)

≤ 4π

Proposition 5.5. Let 0 ≤ a < b. Then


Z b
sin ωt
dt ≤ 2π (5.15)

a t

for ω > 0.

Proof. Define
kπ/ω kπ
sin ωt sin t
Z Z
Ak = dt = dt.
(k−1)π/ω t (k−1)π t

20
Then, again, {Ak } has alternating signs and decreases in absolute value. As
in the previous lemma, we get
Z a
sin ωt
0< dt < A1 .
0 t

Since sint t ≤ 1, we get A1 ≤ π and so
Z b
sin ωt b sin ωt
Z Z a
sin ωt
dt = dt − dt ≤ 2π.

a t 0 t 0 t

The discrete Fourier kernel is a very good approximation of the Dirichlet


kernel.
Proposition 5.6. Let f ∈ L1 (0, π). Let 0 < r ≤ π. Then
sin(n + 12 )t sin(n + 21 )t
Z r Z r
lim f (t) dt = lim f (t) dt (5.16)
n→∞ 0 sin 2t n→∞ 0 t
2

whenever either limit exists.


Proof. By L’Hospital’s rule, we get
 
1 1
lim − = 0.
t→0 sin t t
Define
1 1

 − t 6= 0
g(t) = sin(t/2) t/2
0 t = 0.

Then g is continuous. Thus it is bounded in [0, r] for 0 < r ≤ π and the


function f (t)g(t)χ[0,r](t) is integrable. So are the functions
φ(t) = f (t)g(t) cos(t/2)χ[0,r] (t)
and
ψ(t) = f (t)g(t) sin(t/2)χ[0,r] (t).
Thus,
Z r   Z π Z π
1 1 1
f (t) sin(n+ )t − dt = φ(t) sin nt dt+ ψ(t) cos nt dt
0 2 sin(t/2) t/2 0 0

and, by the Riemann-Lebesgue lemma (cf. Corollary 4.1), both the integrals
on the right-hand side tend to zero as n → ∞ and the result follows.

21
Both the functions Dn (t) and Φn (t) enjoy the Riemann-Lebesgue property.
More precisely, we have the following result.
Proposition 5.7. Let f ∈ L1 (0, π) and let 0 < r ≤ π. Then
Z π Z π
lim f (t)Dn (t) dt = lim f (t)Φn (t) dt = 0. (5.17)
n→∞ r r→∞ r

Proof. Since r > 0, the functions


φ(t) = f (t) sin(t/2)g(t)χ[r,π](t)
ψ(t) = f (t) cos(t/2)g(t)χ[r,π](t)
1 1
where g(t) = sin(t/2) or t/2 , are integrable and the result follows, once again,
from the Riemann-Lebesgue lemma (Corollary 4.1).
Theorem 5.1 (The localisation principle). Let f ∈ L1 (−π, π). Let 0 <
r < π. Then, for x ∈ [−π, π],
Z −r Z π 

lim + f (x − t)Dn (t) dt = 0.
n→∞ −π r

Proof. Fix x ∈ [−π, π]. Then, by the preceding proposition, we have


Z π
f (x − t)Dn (t) dt → 0 as n → ∞.
r

Now, Z −r Z π
f (x − t)Dn (t) dt = f (x + t)Dn (t) dt
−π r
using the evenness of Dn and so once again this integral also tends to zero.
This completes the proof.
Note that the Fourier coefficients depend on the values of a function f
throughout the interval [−π, π]. However, if f and g are in L1 (−π, π) and
for some t ∈ [−π, π] and r > 0, we have f ≡ g in (t − r, t + r) it follows
from the above theorem that the Fourier series of f will converge at t if, and
only if, the Fourier series of g converges at t and in this case the sums of the
Fourier series are the same. Thus the behaviour of the Fourier series at a
point t depends only on the values of the function in a neighbourhood of t.
This is in strong contrast with the behaviour of power series. If two power
series coincide in an open interval, then they are identical throughout their
common domain of convergence.

22
6 Fourier Series of Continuous Functions
A basic question that can be asked is the following: does the Fourier series
of a continuous 2π-periodic function, f , converge to f (t) at every point t ∈
[−π, π]?
Unfortunately, the answer is ‘No!’ and we will study this now.

Proposition 6.1. We have


Z π
lim |Dn (t)| dt = +∞. (6.1)
n→∞ −π

Proof. For t ∈ R, we have | sin t| ≤ |t| and so


π π
| sin(n + 21 )t|
Z Z
|Dn (t)| dt ≥ 4 dt
−π 0 t
(n+ 12 )π
| sin t|
Z
= 4 dt
0 t
n Z kπ
X | sin t|
> 4 dt
k=1 (k−1)π
t
n Z kπ
X | sin t|
> 4 dt

k=1 (k−1)π
n
8 X 1
=
π k=1
k

from which (6.1) follows immediately.

Proposition 6.2. Let V = Cper [−π, π], the space of continuous 2π-periodic
functions with the usual sup-norm (denoted k · k∞ ) and define φn : V → R
by
φn (f ) = sn (f )(0)
where sn (f ) is the nth -partial sum of the Fourier series of f. Then φn is a
continuous linear functional on V and
Z π
1
kφn k = |Dn (t)| dt. (6.2)
2π −π

23
Proof. On one hand,
π
1
Z
φn (f ) = f (t)Dn (t) dt
2π −π

(cf. (5.4d)). Thus,


π
1
Z
|φn (f )| ≤ kf k∞ |Dn (t)| dt
2π −π

and so π
1
Z
kφn k ≤ |Dn (t)| dt.
2π −π

Now, let En = {t ∈ [−π, π] | Dn (t) ≥ 0}. Define

1 − m d(t, En )
fm (t) =
1 + m d(t, En )

where d(t, A) = inf{|t − s| | s ∈ A} is the distance of t from a set A.


Since d(t, A) is a continuous function (in fact, |d(t, A) − d(s, A)| ≤ |t − s|)
fm ∈ Cper [−π, π], (it is periodic since Dn is even and so En is a symmetric
set about the origin). Also kfm k∞ ≤ 1 and fm (t) → 1 if t ∈ En while
fm (t) → −1 if t ∈ Enc . By the dominated convergence theorem, it now
follows that Z π
1
φn (fm ) → |Dn (t)| dt
2π −π
from which (6.2) follows.
Let us now recall a few results from topology and functional analysis.

Theorem 6.1 (Baire). If X is a complete metric space, the intersection of


every countable collection of dense open sets of X is dense in X.

Equivalently, Baire’s theorem also states that a complete metric space


cannot be the countable union of nowhere dense sets.
One of the important consequences of Baire’s theorem is the Banach-
Steinhaus theorem also known as the uniform boundedness principle.

24
Theorem 6.2 (Banach-Steinhaus). Let X be a Banach space and Y a
normed linear space and {Λα }α∈A a collection of bounded linear transforma-
tions from X into Y , where α ranges over some indexing set A. Then either
there exists M > 0 such that

kΛα k ≤ M, for all α ∈ A (6.3)

or,
sup kΛα xk = ∞ (6.4)
α∈A

for all x belonging to some dense Gδ -set in X.

(Recall that a Gδ -set is a set which is the countable intersection of open


sets).
Now consider X = V as defined in Proposition 6.2 and Y = R. Let A = N
and set Λn = φn , again defined in above mentioned proposition. Since, by
Proposition 6.1 and 6.2, we have kφn k → ∞ as n → ∞, it follows from the
Banach-Steinhaus theorem, that there exists a dense Gδ -set (of continuous
2π-periodic functions) in V such that the Fourier series of all these functions
diverge at t = 0. We could have very well dealt with any other point in the
interval [−π, π] in the same manner.
By another application of Baire’s theorem, we can strengthen this further.
Let Ex be the dense Gδ -set of continuous 2π-periodic functions in V such
that the Fourier series of these functions diverge at x. Let {xi } be a countable
set of points in [−π, π] and let

E = ∩ni=1 Exi ⊂ V. (6.5)

Then, by Baire’s theorem E is also a dense Gδ -set. (Each Exi is the countable
intersection of dense open sets and so, the same is true for E). Thus for each
f ∈ E, the Fourier series of f diverges at xi for all 1 ≤ i ≤ ∞. Define

s∗ (f ; x) = sup |sn (f )(x)|.


n

Then s∗ (f, ·) is a lower semi-continuous function. Hence {x | s∗ (f ; x) = ∞}


is a Gδ -set in (−π, π) for each f . If we choose the xi above so that {xi } is
dense (take all rationals, for instance in (−π, π)) then we have the following
result.

25
Proposition 6.3. The set E ⊂ V is a dense Gδ -set such that for all f ∈ E,
the set Qf ⊂ (−π, π) where its Fourier series diverges, is a dense Gδ -set in
(−π, π).

Proposition 6.4. In a complete metric space, which has no isolated points,


no countable dense set can be a Gδ .

Proof. Let E = {x1 , . . . , xn , . . .} be a countable dense set. Assume E is


a Gδ . Thus E = ∩∞ n=1 Wn , Wn open and dense. Then, by hypothesis, Wn \
∪ni=1 {xi } = Vn is also open and dense. But ∩∞ n=1 Vn = ∅, contradicting Baire’s
theorem.
Thus, there exists uncountably many 2π-periodic continuous functions on
[−π, π] whose Fourier series diverge on a dense Gδ -set of (−π, π).
Having answered our first general question negatively, let us now prove a
positive result.

Proposition 6.5. Let f be a 2π-periodic function on [−π, π] which is uni-


formly Lipschitz continuous, i.e. there exists K > 0 such that

|f (x) − f (y) ≤ K|x − y|

for all x, y. Then, the Fourier series of f converges to f on [−π, π].

Proof. Choose 0 < r < π such that

1 r |t/2| ε
Z
dt < .
π −r | sin(t/2)| 2K

t/2
This is possible since sin(t/2) is a bounded continuous function, and hence
integrable on [−π, π]. If C is an upper bound for this function, we need only
επ
choose r such that 2rC < 2K . Let x ∈ [−π, π].
Z π
1
sn (x) = f (x − t)Dn (t) dt.
2π −π

Thus
1 π
Z

|sn (x) − f (x)| = f (x − t) − f (x) Dn (t) dt .
2π −π

26
If r is chosen as above, then, by the localization principle (Theorem 5.1), we
have, for n large enough
Z −r Z π 
1  ε
+ f (x − t) − f (x) D n (t) dt < .
2π −π r
2

On the other hand,


Z r
K r |t|| sin(n + 1/2)t|

1
Z

f (x − t) − f (x) Dn (t) dt ≤
dt

−r 2π −r | sin t/2|
K r |t|/2 ε
Z
≤ dt < .
π −r | sin(t/2)| 2

Thus for n large we have

|sn (x) − f (x)| < ε

which completes the proof.

Corollary 6.1. If f ∈ C 1 [−π, π] is 2π-periodic, then the Fourier series of f


converges to f on [−π, π].

In a later section, we will relax these conditions on f and study the


pointwise convergence of Fourier series.
Remark 6.1. The convergences in Proposition 6.5 and Corollary 6.1 above
are, in fact, uniform over [−π, π], cf. Exercise 32.
Remark 6.2. If f is Lipschitz continuous in a neighbourhood of t ∈ [−π, π],
then we can show, using identical arguments, that the Fourier series of f
converges to f (t) at t.

7 Fejér’s Theorem
In the previous section, we saw that there exist an uncountable number of
continuous 2π-periodic functions whose Fourier series diverge over a dense
set of points. Nevertheless, as we have observed earlier, such functions can be
approximated uniformly by means of trigonometric polynomials over [−π, π],
thanks to the Stone-Weierstrass theorem. Indeed, we can now prove the
following result.

27
Theorem 7.1 (Fejér). Let f be a continuous 2π-periodic function on
[−π, π]. Let {sn }n≥0 be the sequence of partial sums of its Fourier series.
Define
s0 (x) + . . . + sn (x)
σn (x) = .
n+1
Then σn → f uniformly over [−π, π].
Proof. It is immediate, from the definition of the Fejér kernel Kn , to see that
Z π
1
σn (x) = f (x − t)Kn (t) dt.
2π −π
Thus π
1
Z

σn (x) − f (x) = f (x − t) − f (x) Kn (t) dt,
2π −π

in view of (5.11). Now, since f is continuous on [−π, π], it is bounded and


uniformly continuous. Let |f (x)| ≤ M for all x ∈ [−π, π] and, given ε > 0,
let δ > 0 such that |x − y| < δ implies |f (x) − f (y)| < ε/2.
Now, choose N large enough such that, for n ≥ N, Kn (t) ≤ ε/4M for
all δ < |t| ≤ π. This is possible because of the estimate (5.12). Thus, since
Kn ≥ 0,
Z δ
ε π
Z

f (x − t) − f (x) Kn (t) dt ≤
Kn (t) dt = πε,

−δ 2 −π
again by (5.11). On the other hand,
Z −δ Z π 
≤ 2M · ε · 2π = πε

+ f (x − t) − f (x) K n (t) dt

−π δ
4M
if n ≥ N. It now follows that for all x ∈ [−π, π], we have

|σn (x) − f (x)| < ε

for n ≥ N which completes the proof.


Corollary 7.1. Let f and g be two 2π-periodic and continuous functions on
[−π, π]. If they have the same Fourier series, then f ≡ g.
Proof. If the two functions have the same Fourier series, then the σn will be
the same for both functions and we know that σn → f and σn → g uniformly
on [−π, π]. Hence the result.

28
P
Given a series n an , we say that it is Cesàro summable or (C, 1)
summable to a if σn → a as n → ∞, where
s1 + . . . + sn
σn = ,
n
sk being the partial sums of the series. Thus the Fourier series of a continuous
2π-periodic function is always Cesàro summable to the function.
Thus if f is a continuous 2π-periodic function whose Fourier series is given
by

a0 X
+ (an cos nt + bn sin nt),
2 n=1

then f is uniformly approximated over [−π, π] by the trigonometric polyno-


mials n  
a0 X k 
σn (x) = + 1− ak cos kt + bk sin kt . (7.1)
2 n+1
k=1

Starting from this, we can deduce Weierstrass’ approximation theorem. In-


deed, let f ∈ C[−1, 1]. Define g(t) = f (cos t), for t ∈ [−π, π]. Then g is
2π-periodic and continuous. Further, g is an even function and hence its
Fourier series will only consist of cosine terms. Let the Fourier series of g be
given by

a0 X
+ ak cos kt.
2 k=1

Then n  
a0 X k
σn (t) = + 1− ak cos kt.
2 k=1
n+1
Thus, given ε > 0, there exists N such that, for all n ≥ N,

n  
a0
X k
f (cos t) − − 1− ak cos kt < ε

2 k=1
n+1

for all t ∈ [−π, π]. This is the same as



n  
a0 X k −1

f (t) − − 1− ak cos k(cos t) < ε

2 k=1
n+1

29
for all t ∈ [−1, 1]. Now it only remains to show that

Pk (t) = cos(k cos−1 t)

is a polynomial in t for every non-negative integer k. Indeed, P0 (t) ≡ 1 and


P1 (t) ≡ t. Assume that Pk (t) is a polynomial in t, of degree k, for every
1 ≤ k ≤ n − 1, n ≥ 2. Then

cos ns = cos ns + cos(n − 2)s − cos(n − 2)s


 
= cos (n − 1)s + s + cos (n − 1)s − s − cos(n − 2)s
= 2 cos(n − 1)s cos s − cos(n − 2)s
= 2 cos s Pn−1 (cos s) − Pn−2 (cos s).

It then follows that


Pn (t) = 2tPn−1 (t) − Pn−2 (t) (7.2)
and so Pn (t) is a polynomial of degree n in t.
The polynomials {Pn } defined recursively via (7.2) where P0 ≡ 1 and
P1 (t) = t, are called the Chebychev polynomials and play an important
role in numerical analysis, especially in numerical quadrature.

8 Regularity
In this section, we will briefly discuss various regularity assumptions to be
made on functions when discussing the pointwise convergence of Fourier se-
ries.
Let f : [a, b] → R be a given function which is differentiable on (a, b).
Assume that
f 0 (a+) = lim f 0 (t)
t→a
0
exists. Then f is bounded in a subinterval (a, a+δ) (where δ > 0) and so, by
the mean-value theorem, f is uniformly continuous on (a, a + δ). Thus it can
be continuously extended to [a, a + δ]. Consequently f (a+) = limt→a f (t),
exists.
Again by the mean-value theorem, applied to f˜ where f˜(a) = f (a+) and
˜
f (t) = f (t) for a < t ≤ a + δ, we have

f (t) = f (a+) + f 0 (a + θ(t − a))(t − a)

30
where 0 < θ < 1. Thus
f (t) − f (a+)
lim = f 0 (a+). (8.1)
t→a t−a
Notice that f 0 (a+) is different from the right-sided derivative of f at a (if it
exists), which is given by

f (t) − f (a)
D + f (a) = lim .
t↓a t−a

We also define, f 0 (b−) = limt→b f 0 (t).

Definition 8.1. We say that a function f : [a, b] → R is piecewise smooth


if there exist a finite number of points

a = a0 < a1 < . . . < an = b

such that f is continuously differentiable in each subinterval (ak , ak+1 ), 0 ≤


k ≤ n − 1 and f 0 (c+) and f 0 (c−) exist at all points c ∈ [a, b] except at a,
where f 0 (a+) exists, and at b, where f 0 (b−) exists.

Let f : [a, b] → R (or C) be a given function. Let P = {a = x0 < x1 <


. . . < xn = b} be any partition of [a, b]. Define
n
X
V (P; f ) = |f (xi ) − f (xi−1 )|
i=1

and
V (f ; a, b) = sup V (P; f )
P

the supremum being taken over all possible partitions of [a, b]. The quantity
V (f ; a, b) is called the total variation of f over the interval [a, b].

Definition 8.2. A real (or complex) valued function defined on [a, b] is said
to be of bounded variation on [a, b] if V (f ; a, b) < ∞.

Example 8.1. Any monotonic function defined on [a, b] is of bounded varia-


tion. In this case
V (f ; a, b) = |f (b) − f (a)|.

31
Example 8.2. If f is uniformly Lipschitz continuous, then it is of bounded
variation. For,
X X
|f (xi ) − f (xi−1 )| ≤ L (xi − xi−1 ) = L(b − a) < ∞
i i

Example 8.3. Define


(
x2 sin x12 , 0 < x < 1
f (x) =
0, x = 0.

Then, f is not of bounded variation. To see this, choose a partition of [0, 1]


as follows: (s )
2
{0, 1} ∪ |0≤j≤n .
π(2j + 1)

2 2 2 4j
|f (xj ) − f (xj−1)| = + = .
π(2j + 1) π(2j − 1) π 4j 2 − 1
2 4j 2
≥ 2
= .
π 4j πj
Thus for all n, V (f ; 0, 1) ≥ π2 nj=1 1j and so V (f ; 0, 1) = ∞.
P

Given a real number r, define r + = max{r, 0} and r − = − min{r, 0}.


Then r = r + − r − and |r| = r + + r − . Thus, if f : [a, b] → R, and P = {a =
x0 < . . . < xn = b} is any partition, set
n
X +
p(P; f ) = f (xi ) − f (xi−1 )
i=1

and n
X −
n(P; f ) = f (xi ) − f (xi−1 ) .
i=1

Then V (P; f ) = p(P; f ) + n(P; f ) and


f (b) − f (a) = p(P; f ) − n(P; f ).
Define
P (f ; a, b) = sup p(P; f ) and N(f ; a, b) = sup n(P; f );
P P

where, again, the supremum are taken over all possible partitions of [a, b].

32
Proposition 8.1. Let f be of bounded variation on [a, b]. Then

V (f ; a, b) = P (f ; a, b) + N(f ; a, b) (8.2)

f (b) − f (a) = P (f ; a, b) − N(f ; a, b). (8.3)

Proof. We know that, for any partition P,

p(P; f ) = n(P; f ) + f (b) − f (a)


≤ N(f ; a, b) + f (b) − f (a).

Then, taking the supremum over all possible partitions, we get

P (f ; a, b) ≤ N(f ; a, b) + f (b) − f (a). (8.4)

Similarly n(P; f ) = p(P; f ) + f (a) − f (b) yields

N(f ; a, b) ≤ P (f ; a, b) + f (a) − f (b) (8.5)

Relations (8.4) and (8.5) yield (8.3), since V (f ; a, b) < ∞ implies that
P (f ; a, b) < ∞ and N(f ; a, b) < ∞. Now

V (P; f ) = p(P; f ) + n(P; f )

gives us
V (f ; a, b) ≤ P (f ; a, b) + N(f ; a, b). (8.6)
On the other hand,

V (f ; a, b) ≥ p(P; f ) + n(P; f )

= 2p(P; f ) − f (b) − f (a)
= 2p(P; f ) + N(f ; a, b) − P (f ; a, b)

using (8.3). Again, taking the supremum over all partitions, we get

V (f ; a, b) ≥ 2P (f ; a, b) + N(f ; a, b) − P (f ; a, b)
= P (f ; a, b) + N(f ; a, b) (8.7)

Relations (8.6) and (8.7) yield (8.2).

33
Proposition 8.2. If f and g are of bounded variation on [a, b], then f + g
is of bounded variation and
V (f + g; a, b) ≤ V (f ; a, b) + V (g; a, b). (8.8)
Proof. If P is any partition,
V (P; f + g) ≤ V (P; f ) + V (P; g)
by the triangle inequality and the result follows.
We now have an important characterization of functions of bounded vari-
ation.
Theorem 8.1. A function f : [a, b] → R is of bounded variation if, and only
if, f is the difference of two monotonic increasing functions.
Proof. If f is the difference of two monotonic increasing functions, since
each of these is of bounded variation (cf. Example 8.1), it follows from the
preceding proposition that f is of bounded variation.
Conversely, if f is of bounded variation on [a, b], by Proposition 8.1, we
have, for any x ∈ [a, b],
f (x) − f (a) = P (f ; a, x) − N(f ; a, x).
Let g(x) = P (f ; a, x) and h(x) = N(f ; a, x)−f (a). Clearly both functions are
monotonic increasing and f (x) = g(x) − h(x). This completes the proof.
From the properties of monotonic functions, we can now deduce the fol-
lowing result.
Corollary 8.1. If f : [a, b] → R is of bounded variation, then f is dif-
ferentiable a.e. Further, for all x ∈ (a, b), f (x+) and f (x−) exist. So do
f (a+) and f (b−). The function f has at most a countable number of jump
discontinuities.
The fundamental theorem of Lebesgue integration states that if f is in-
tegrable on [a, b] and if
Z x
F (x) = c + f (t) dt,
a

then F 0 = f a.e. on [a, b]. We now ask the question as to when a function
can be expressed as an indefinite integral of an integrable function.

34
Proposition 8.3. Let f be integrable on [a, b]. Then set
Z x
F (x) = f (t) dt.
a

Then F is of bounded variation on [a, b].


Proof. If P = {a = x0 < x1 < . . . < xn = b} is any partition of [a, b], then
X n n Z xi
X Z b
|F (xi ) − F (xi−1 )| ≤ |f (t)| dt = |f (t)| dt < ∞.
i=1 i=1 xi−1 a

Thus Z b
V (F ; a, b) ≤ |f (t)| dt < ∞.
a
Hence the result.
Thus a function must be at least of bounded variation on [a, b] for it to
be expressed as an indefinite integral of an integrable function. But this is
not enough.
Proposition 8.4. Let f : [a, b] → R be integrable. Given ε > 0, there exists
δ > 0 such that if E ⊂ [a, b] is a measurable set with µ(E) < δ, then
Z

f (x) dx < ε.

E

Proof. If |f | ≤ M on [a, b], the result is trivially true, since


Z

f dx ≤ Mµ(E),

E

where µ(E) is the Lebesgue measure of E.


In the general case, define
(
|f (x)|, if |f (x)| ≤ n
fn (x) =
n, if |f (x)| > n.

Then fn is bounded and fn → |f | pointwise. Further {fn } is an increasing


sequence of non-negative functions. By the monotone convergence theorem,
Z b Z b
lim fn dx = |f | dx < ∞.
n→∞ a a

35
Given ε > 0, choose N such that
Z b
(|f | − fN ) dx < ε/2.
a

Now choose δ > 0 such that µ(E) < δ implies that


Z
fN dx < ε/2.
E
R R Rb R
Then | E f dx| ≤ E |f | dx ≤ a (|f | − fN ) dx + E fN dx < ε. This completes
the proof.

Definition 8.3. A function f : [a, b] → R is said to be absolutely contin-


uous on [a, b] if for every ε > 0, there exists δ > 0 such that whenever we
have a finite collection of disjoint intervals {(xi , x0i )}ni=1 satisfying
n
X
(x0i − xi ) < δ
i=1

we have n
X
|f (x0i ) − f (xi )| < ε.
i=1

Clearly, any absolutely continuous function is uniformly continuous on


[a, b].
Example 8.4. Any uniformly Lipschitz continuous function is absolutely con-
tinuous, since
X X
|f (x0i ) − f (xi )| ≤ L (x0i − xi ) < Lδ.

Example 8.5. Any indefinite integral of an integrable function is absolutely


continuous by virtue of Proposition 8.4.
We will show presently that a function can be written as an indefinite
integral if, and only if, it is absolutely continuous.

Proposition 8.5. An absolutely continuous function is of bounded variation.

36
Proof. Let δ correspond to ε = 1 in the definition of absolute continuity. Let
K be the integral part of 1 + (b − a)/δ, where [a, b] is the given interval.
Given any partition P of [a, b], we can refine it to a partition consisting of K
sets of sub-intervals each of total length less than δ. Thus V (P; f ) ≤ K and
so V (f ; a, b) ≤ K.
Consequently, any absolutely continuous function is differentiable a.e.. on
[a, b].
Proposition 8.6. Let f : [a, b] → R be absolutely continuous. Assume that
f 0 = 0 a.e. in [a, b]. Then f is a constant function.
Proof. Let c ∈ (a, b). Let

E = {x ∈ (a, c) | f 0 (x) = 0}

Then µ(E) = c − a. Let ε, η > 0 be arbitrary. If x ∈ E, there exists a


sufficiently small h > 0 such that [x, x+h] ⊂ [a, c] and |f (x+h)−f (x)| < ηh.
By the Vitali covering lemma, there exists a finite disjoint collection of such
intervals which cover all of E except possibly a subset of measure less than
δ, where δ corresponds to ε in the definition of absolute continuity. We label
these intervals [xk , yk ], with xk increasing. Thus

y0 = a ≤ x1 < y1 ≤ x2 < y2 ≤ . . . ≤ yn = c = xn+1 .

Then,
n
X
|xk+1 − yk | < δ.
k=0

Now,
n
X X
|f (yk ) − f (xk )| < η (yk − xk ) < η(c − a).
k=1

Also n
X
|f (xk+1 ) − f (yk )| < ε
k=1

by absolute continuity. Thus

|f (c) − f (a)| ≤ ε + η(c − a)

or f (c) = f (a) for all c ∈ (a, b). Hence the result.

37
Theorem 8.2. A function can be expressed as an indefinite integral of an
integrable function if, and only if, it is absolutely continuous. The derivative
of this function (which exists a.e.) is equal a.e. to the integrand.

Proof. If f were an indefinite integral, it is absolutely continuous (cf. Exam-


ple 8.5).
Conversely, let F : [a, b] → R be absolutely continuous. Then it is of
bounded variation and

F (x) = F1 (x) − F2 (x)

where Fi (x), i = 1, 2 are monotonic increasing. Thus F 0 = F10 − F20 a.e. and
F10 ≥ 0, F20 ≥ 0. Thus,
Z b Z b Z b
0
|F | dx ≤ + |F20 | dx
|F10 | dx
a a a
Z b Z b
= F10 dx + F20 dx
a a
≤ F1 (b) − F1 (a) + F2 (b) − F2 (a) < ∞.
Rx
Thus F 0 is integrable. Now let G(x) = a F 0 (t) dt. Then G, being an indefi-
nite integral of an integrable function, is absolutely continuous and G0 = F 0
a.e. Thus G − F is absolutely continuous and (G − F )0 = 0 a.e.. Thus, by
Proposition 8.6, (G − F )(x) = (G − F )(a) for all x ∈ [a, b]. Hence,
Z x
F (x) = F (a) + F 0 (t) dt.
a

This completes the proof.


Absolutely continuous functions share many properties of continuously
differentiable functions. In particular, we have the following result.

Theorem 8.3 (Integration by parts). Let f, g be absolutely continuous


on [a, b]. Then
Z b Z b
0
f (t)g (t) dt = f (b)g(b) − f (a)g(a) − f 0 (t)g(t) dt. (8.9)
a a

38
Proof. Consider the function f 0 (x)g 0 (y) on [a, b]×[a, b]. Consider the integral
Z bZ x
f 0 (x)g 0 (y) dy dx.
a a

It is a routine verification to check that Fubini’s theorem applies and so


Z bZ x Z bZ b
0 0
f (x)g (y) dy dx = f 0 (x)g 0 (y) dx dy.
a a a y

The left-hand side gives us


Z b Z x  Z b
0 0
g(x) − g(a) f 0 (x) dx

g (y) dy f (x) dx =
a a a
Z b
g(x)f 0(x) dx − g(a) f (b) − f (a)
 
=
a

by repeated use of Theorem 8.2. The right-hand side gives


Z b Z b  Z b
0 0
f (b) − f (y) g 0 (y) dy

f (x) dx g (y) dy =
a y a
Z b
f (y)g 0(y) dy.
 
= f (b) g(b) − g(a) −
a

Equating the two, we deduce (8.9). This completes the proof.

9 Pointwise Convergence
In this section, we will prove the convergence theorems of Dirichlet and Jor-
dan.
Proposition 9.1. Let f ∈ L1 (0, π) and assume that f 0 (0+) exists. Then

1 π sin(n + 12 )t 1
Z
lim f (t) dt = f (0+). (9.1)
n→∞ π 0 t 2
Proof. Adding and subtracting f (0+) in the integrand, we get

sin(n + 21 )t sin(n + 12 )t sin(n + 21 )t


Z π Z π Z π
f (t) dt = (f (t)−f (0+)) dt+f (0+) dt.
0 t 0 t 0 t

39
The second integral on the right-hand side becomes (after a change of vari-
able),
Z (n+ 1 )π
2 sin t
dt
0 t
which converges to π/2 as n → ∞. Thus,

1 π
sin(n + 21 )t 1
Z
lim f (0+) dt = f (0+).
n→∞ π o t 2
Hence we need to show that the first term tends to zero as n → ∞. Let ε > 0
be an arbitrarily small number. Choose 0 < r < π such that for 0 < t ≤ r,

f (t) − f (0+) 0

− f (0+) < ε
t

( cf. (8.1)). Then,


sin(n+ 21 )t
 R
r
Z π
sin(n + 21 )t  0 (f (t) − f (0+))

t
dt
(f (t) − f (0+)) dt =
0 t  Rπ sin(n+ 12 )t
+ (f (t) − f (0+)) dt.

r t

The second term tends to zero, by the Riemann-Lebesgue property for Φn


(cf. Proposition 5.7), since f (t) − f (0+) ∈ L1 (0, π). Now, the first term
above can be written as
Z r Z r
f (t) − f (0+)

0 1 0 1
− f (0+) sin(n + )t dt + f (0+) sin(n + )t dt.
0 t 2 0 2

But,
Z r  
f (t) − f (0+) 0 1
− f (0+) sin(n + )t dt < rε < πε

0 t 2

and
r r r
1 t t
Z Z Z
sin(n + )t dt = sin cos nt dt + cos sin nt dt
0 2 0 2 0 2
and both these integrals tend to zero, as n → ∞, by the Riemann-Lebesgue
lemma (cf. Corollary 4.1). This completes the proof.

40
Theorem 9.1. (Dirichlet) Let f ∈ L1 (−π, π) be a piecewise smooth func-
tion. Then its Fourier series converges to 12 (f (t+) + f (t−)) at all points
t ∈ [−π, π]. In particular, if f is continuous at a point t, then its Fourier
series converges to f (t) at that point.

Proof. Recall that (cf. (5.4e))


Z π
1
sn (t) = (f (t + x) + f (t − x))Dn (x) dx.
2π 0

Now, by (9.1), we have

1 π
sin(n + 21 )x 1
Z
lim f (t + x) dx = f (t+).
n→∞ π 0 x 2
Thus,
1 π
sin(n + 21 )x 1
Z
lim f (t + x) x dx = f (t+).
n→∞ 2π 0 2
2
Hence, by Proposition 5.6, we get

sin(n + 21 )x
Z π
1 1
lim f (t + x) x dx = f (t+).
n→∞ 2π 0 sin 2 2

Similarly,
1 π
sin(n + 21 )x 1
Z
lim f (t − x) x dx = f (t−)
n→∞ 2π 0 sin 2 2
from which the result follows.
Example 9.1. Consider the function

 −1, −π ≤ t < −π/2
f (t) = 0, −π/2 ≤ t ≤ π/2
1, π/2 < t ≤ π.

This function is piecewise smooth and is odd. Thus, its Fourier series consists
only of sine functions. Now,
1 π 2 π
Z Z
bn = f (t) sin nt dt = sin nt dt.
π −π π π2

41
Thus,
2

 n odd

,
bn = 0, n = 4k, k ∈ N
4
− nπ , n = 4k + 2, k ∈ N

At t = π/2, the series must thus converge to 1/2. Thus,


 
1 2 1 1 1
= 1− + − +···
2 π 3 5 7

which yields the well known Gregory series


π 1 1 1
= 1− + − +···
4 3 5 7
In order to prove the next result, we recall a version of the mean value
theorem for integrals.

Proposition 9.2. Let g : [a, b] → R be continuous and let f : [a, b] → R


be non-negative and monotonic increasing. Then, there exists c ∈ [a, b] such
that Z b Z b
f (t)g(t) dt = f (b) g(t) dt. (9.2)
a c

Remark 9.1. The result is false if f is not non-negative. To see this, take
f (t) = g(t) = t on [−1, 1]. Then the left-hand side will be
Z 1
2
t2 dt = .
−1 3

The right-hand side is


1
1 − c2
Z
f (1) t dt =
c 2

and we can never have c ∈ [−1, 1] such that the two are equal.

Remark 9.2. In the mean value theorem of differential calculus, the point
c will be in the interior of the interval. In case of mean value theorems for
integrals, this need not be necessarily the case. For instance, if f ≡ 1 on
[a, b] and if g is strictly positive on that interval, then we cannot have (9.2)
with c ∈ (a, b).

42
We are now in a position to prove a convergence theorem for a larger class
of functions. We need a preliminary result analogous to Proposition 9.1.
Proposition 9.3. Let f be of bounded variation on [0, π]. Then
1 π sin(n + 12 )t 1
Z
lim f (t) dt = f (0+). (9.3)
n→∞ π 0 t 2
Proof. Since f is of bounded variation, it can be written as the difference of
two monotonic increasing functions. Hence we may assume, without loss of
generality, that f is monotonic increasing. Now,
sin(n+ 21 )t
 R
π
Z π 1
sin(n + 2 )t

 0
(f (t) − f (0+)) t
dt
f (t) dt =
0 t  R π sin(n+ 21 )t
+ f (0+) 0 dt.

t
As already shown in the proof of Proposition 9.1, the second integral con-
verges to π/2. Thus, as before, it is enough to show that the first integral
tends to zero as n → ∞.
Let ε > 0 be an arbitrarily small number. Choose 0 < r ≤ π such that
|f (t) − f (0+)| < ε/4, 0 < t ≤ r. (9.4)
Then, splitting the first integral over [0, r] and [r, π], we see that the integral
on [r, π] tends to zero by the Riemann-Lebesgue property for Φn (cf. Propo-
sition 5.7) since f (t) − f (0+) is integrable over that interval. Thus, for n
sufficiently large, we have
Z π 1
1 sin(n + 2
)t ε

π (f (t) − f (0+)) dt < .
r t 2
On the interval [0, r], we redefine the function f as f (0+) at t = 0, so that
the function f (t) − f (0+) remains non-negative and monotonic increasing
without altering the value of the integral over that interval. Hence, by the
mean value theorem (cf. Proposition 9.2), there exists c ∈ [0, r] such that
sin(n + 12 )t sin(n + 21 )t
Z r Z r
(f (t) − f (0+)) dt = (f (r) − f (0+)) dt.
0 t c t
Thus, by Proposition 5.5 and by (9.4), it follows that
Z r 1
1 sin(n + 2
)t ε

π (f (t) − f (0+)) dt < .
0 t 2
This completes the proof.

43
Theorem 9.2. (Jordan) Let f be a function of bounded variation on [−π, π].
Then, its Fourier series at any point t in this interval converges to 21 (f (t+) +
f (t−)). In case f is continuous at t, then the series converges to f (t).
Proof. The proof is identical to that of Theorem 9.1, except that we appeal
to Proposition 9.2 in place of Proposition 9.1.
Remark 9.3. Dirichlet proved his theorem on the convergence of Fourier
series in 1829. Jordan’s result was proved in 1881. In 1904, Fejér showed
that if f ∈ L1 (−π, π), and if f (t+) and f (t−) exist at t ∈ [−π, π], then
the Fourier series is (C, 1)-summable at t to (f (t+) + f (t−))/2. One of the
greatest triumphs in the history of Fourier series is the result of Carlesson
(1966) that the Fourier series of any function in L2 (−π, π) converges to the
value of that function a.e..This was extended by Hunt in 1968 to all functions
in Lp (−π, π) for 1 < p < ∞.

10 Termwise Integration
P
In general, when we have a series f (x) = n fn (x), we can integrate the
series term-by-term if the series is uniformly convergent. However, we have
seen that Fourier series (under appropriate hypotheses) converge to (f (t+) +
f (t−))/2 at a discontinuity. Thus, if the function is discontinuous, uniform
convergence is ruled out and the above principle does not apply. However,
Fourier series are special and enjoy special properties. Vis-à-vis integration,
we have the following result.
Theorem 10.1. Let f ∈ L1 (−π, π) be extended periodically over R and have
the Fourier series ∞
a0 X
+ (an cos nt + bn sin nt).
2 n=1
Then,
(i) the series obtained by termwise integration, viz.
∞  
a0 X an bn
x+ sin x − cos x + C
2 n=1
n n

where ∞
X bn
C = ,
n=1
n

44
Rx
converges to 0 f (t) dt.
(ii) This convergence is uniform if f ∈ L2 (−π, π).

Proof. Let
x
a0 
Z 
F (x) = f (t) − dt.
0 2
Since f is integrable, it follows that F is absolutely continuous. It is also
2π-periodic. For,
Rx R x+2π
F (x + 2π) = 0 f (t) − a20 dt + x f (t) − a20 dt
 

Rπ a0

= F (x) + −π
f (t) − 2
dt

since, by 2π-periodicity, the integration can be done over any interval of


length 2π without affecting its value. But the last integral vanishes by the
definition of a0 . Thus F (x + 2π) = F (x).
Since it is absolutely continuous, it is continuous and of bounded vaiation
and so its Fourier series converges to its value at each point. Let

c0 X
F (t) = + (cn cos nt + dn sin nt).
2 n=1

We can also use integration by parts (available for absolutely continuous


functions (cf. (8.9)) to obtain

cn = π1 −π F (t) cos nt dt

1
π 1
Rπ a0

= nπ
F (t) sin nt −π − nπ −π
f (t) − 2
sin nt dt.
a0
The boundary terms vanish by periodicity. So does the integral of 2
sin nt.
Thus we deduce that cn = −bn /n. Similarly, dn = an /n. Thus,
∞  
c0 X an bn
F (t) = + sin nt − cos nt .
2 n=1
n n

Evaluating this at t = 0, we get



c0 X bn
= .
2 n=1
n

45
This proves (i).
If f ∈ L2 (−π, π), then the series n a2n and n b2n are convergent, by
P P
virtue of Parseval’s P
identity (cf. (2.7)).
P Hence, by the Cauchy-Schwarz in-
equality, the series n an /n and n bn /n are absolutely convergent. Thus,
by the Weierstrass’ M-test, the above series for F (t) is uniformly conver-
gent.

Remark 10.1. Since we have shown that



c0 X bn
= ,
2 n=1
n

1
P for any f ∈ L (−π, π), the Fourier sine coefficients bn are
it follows that
such that n bn /n is convergent. The absolute convergence of this series,
however, is true under the additional hypothesis that f ∈ L2 (−π, π).

Remark 10.2. If f is continuous and 2π-periodic, then it is in L2 (−π, π) and


so, for any continuous 2π-periodic function, the termwise integrated Fourier
series is uniformly convergent.

We now give an application of the observation made in Remark 10.1 by


presenting an example of a uniformly convergent trigonometric series which
cannot be a Fourier series.
Example 10.1. Consider the trigonometric series

X sin nt
. (10.1)
n=2
log n

The sequence {(log n)−1 } is non-negative and monotonically decreases to


zero. Further,for t 6= 2nπ,
n
X cos 2t − cos(n + 21 )t
sin kt =
k=1
2 sin 2t

and so
n
X 1 1
sin kt ≤ ≤

t
| sin 2 | sin a2


k=1

46
for t ∈ [a, 2π −a] for any a ∈ (0, π). Thus, by Dirichlet’s test, the given series
in (10.1) converges uniformly in [a, 2π − a] for any a ∈ (0, π). However, the
series ∞ ∞
X bn X 1
=
n=2
n n=2
n log n
is divergent and so the given series cannot be a Fourier series.
P
Remark 10.3. Note that in a Fourier series, it is necessary
P that n bn /n
is convergent. However, there is no such condition on n an /n. Indeed,
Stromberg (1981) has shown that the series

X cos nt
n=2
log n

is a Fourier series!

11 Termwise Differentiation
Consider the Fourier series of the 2π-periodic extension of the function f (t) =
t on [−π, π]. Of course, such an extension has jump discontinuities at all odd
multiples of π. Nevertheless,

X (−1)n+1
f (t) = 2 sin nt
n−1
n

and, by Jordan’s theorem, the series converges to f (t) at all points in (−π, π)
and to zero at −π and π. The termwise derivative of the series is

X
2 (−1)n+1 cos nt
n=1

which is divergent everywhere since cos nt does not converge to zero, as n →


∞, for any t.
Thus, while any Fourier series may be integrated termwise meaningfully,
extra hypotheses are needed for termwise differentiation.

47
Theorem 11.1. Let f be a continuous and piecewise smooth 2π-periodic
function on [−π, π] with Fourier series

a0 X
+ (an cos nt + bn sin nt).
2 n=1

If its derivative f 0 is also piecewise smooth, then



f 0 (t+) + f 0 (t−) X
= (nbn cos nt − nan sin nt). (11.1)
2 n=1

Proof. Let f 0 have the Fourier series expansion



c0 X
+ (cn cos nt + dn sin nt).
2 n=1

Since f is 2π-periodic, we have


π
1
Z
c0 = f 0 (t) dt = 0.
π −π

Again, for n ≥ 1,
1

cn = π −π
f 0 (t) cos nt dt

1 π
n

= π
f (t) cos nt −π + π −π
f (t) sin nt dt,
by the absolute continuity of f , which is piecewise smooth. The boundary
terms cancel out by the 2π-periodicity and so we get cn = nbn . Similarly, dn =
−nan . The relation (11.1) follows from the Dirichlet convergence theorem
(Theorem 9.1) for piecewise smooth functions.
Remark 11.1. By the Riemann-Lebesgue lemma, cn and dn tend to zero as
n → ∞. Thus, nan and nbn tend to zero. More generally, the smoother the
function, the faster its Fourier coefficients tend to zero. If f, f 0 , f 00 , · · · f (k−1)
are all continuous and f (k) is piecewise smooth, then nk an and nk bn tend
to zero as n → ∞. Also, in this case, since f (k) ∈ L2 (−π, π), we have, by
Parseval’s identity, that

X ∞
X
n2k a2n < ∞, and n2k b2n < ∞.
n=1 n=1

48
We conclude with a nice appliation of Fourier series to a problem in the
calculus of variations.
One of the earliest known problems in the calculus of variations is known
as the classical isoperimetric problem and can be traced to Virgil’s Aeneid,
which describes the problem of Dido, future queen of Carthage. In mathe-
matical terms, the problem can be stated as follows.
• Of all simple closed curves in the plane with a fixed length, which one
encloses the maximum area?
Or, equivalently:
• Of all simple closed curves in the plane enclosing a fixed area, which
one has the least length?
Given the existence of such a curve, it is possible to give a fairly elemen-
tary geometric argument to identify it as the circle. However, the existence
of an optimal solution requires extra proof.
We can deal with the existence and the uniqueness in one stroke if we
prove the classical isoperimetric inequality: if L is the length of a simple
closed plane curve and if A is the enclosed area, then
L2 ≥ 4πA, (11.2)
with equality if, and only if, the curve is a circle. For the circle, we do indeed
have equality since L = 2πr and A = πr 2 , where r is its radius. On the other
hand, for any curve of given length L, the maximum possible value for the
enclosed area is L2 /4π, which is achieved for the circle. Thus, this establishes
the circle as the optimal solution. The uniqueness follows from the ‘only if’
part of the proof.
Let us assume that a simple closed curve C is parametrized by the equa-
tions x = x(s), y = y(s), where s is the arc length, which varies over the
interval [0, L]. We assume that the functions x(s) and y(s) verify the hy-
potheses of Theorem 11.1. We reparametrize the equations using the param-
eter

t = s
L
so that t ∈ [0, 2π] and x and y are 2π-periodic smooth functions in t with
piecewise smooth derivaives. Let
x(t) = a20 + ∞
P 
n=1 (an cos nt + bn sin nt) 
P∞ (11.3)
c0
y(t) = 2 + n=1 (cn cos nt + dn sin nt).

49
Thus, P∞
x0 (t) = n=1 (nbn cos nt − nan sin nt)
P∞ (11.4)
0
y (t) = n=1 (ndn cos nt − ncn sin nt).
Since  2  2
dx dy
+ = 1,
ds ds
we get that 2  2
  2
dx dy L
+ = .
dt dt 2π
Then, by Parseval’s identity, it follows that
 2 Z " 2  2 # ∞
L 1 2π dx dy X
2 = + dt = n2 (a2n + b2n + c2n + d2n ).
2π π 0 dt dt n=1
(11.5)
Now, the enclosed area A is given by
Z 2π ∞
dy
Z X
A = x dy = x(t) (t) dt = π n(an dn − bn cn ). (11.6)
C 0 dt n=1

From (11.5) and (11.6), we get



X
2 2
(nan − dn )2 + (nbn + cn )2 + (n2 − 1)(c2n + d2n )
 
L − 4πA = 2π
n=1

which is non-negative, thus proving (11.2).


If equality occurs in (11.2), then the above relation shows that for all
n ≥ 1,
nan = dn and nbn = −cn .
Also, it shows that, for n > 1, cn = dn = 0, and hence it follows that
an = bn = 0 as well for those n. Thus,
a0
x(t) = 2
+ a1 cos t + b1 sin t

c0
y(t) = 2
− b1 cos t + a1 sin t
from which we get
 a0 2  c0 2
x(t) − + y(t) − = a21 + b21
2 2
50
and so the curve has to be a circle, thus proving the uniqueness of the optimal
solution.
The isoperimetric inequality exists in all dimensions. The n-dimensional
ball is the unique domain with given ‘(n − 1)-dimensional surface area’ and
maximizing the enclosed (n-dimensional) volume amongst all possible do-
mains.
In three dimensions, the analogue of (11.2) reads as

S 3 ≥ 36πV 2

where S is the surface area and V is the enclosed volume.


If Ω ⊂ Rn is a bounded domain, then let us denote its n-dimensional
(Lebesgue) measure by |Ω|n and the ‘(n − 1)-dimensional surface measure’ of
the boundary ∂Ω (which has to be suitably defined) by |∂Ω|n−1 . The classical
isoperimetric inequality now reads as
1 1
1− n
|∂Ω|n−1 ≥ nωnn |Ω|n

where n
π2
ωn = n
Γ( 2 + 1)
is the volume of the unit ball in Rn . Equality occurs in the isoperimetric
inequality if, and only if, Ω is a ball.

51
References
[1] Bachman, G., Narici, L. and Beckenstein, E. Fourier and Wavelet
Analysis, Springer.

[2] Churchill, R. V. Fourier Series and Boundary Value Problems,


McGraw-Hill.

[3] Royden, H. L. Real Analysis, Macmillan.

[4] Rudin, W. Principles of Mathematical Analysis, McGraw-Hill.

[5] Rudin, W. Real and Complex Analysis, Tata McGraw-Hill.

[6] Simmons, G. F. Introduction to Topology and Modern Analysis,


McGraw-Hill.

52

Vous aimerez peut-être aussi